Boshernitzan’s condition, factor complexity, and an application

By Van Cyr and Bryna Kra

Abstract

Boshernitzan gave a decay condition on the measure of cylinder sets that implies unique ergodicity for minimal subshifts. Interest in the properties of subshifts satisfying this condition has grown recently, due to a connection with discrete Schrödinger operators, and of particular interest is how restrictive the Boshernitzan condition is. While it implies zero topological entropy, our main theorem shows how to construct minimal subshifts satisfying the condition, and whose factor complexity grows faster than any pre-assigned subexponential rate. As an application, via a theorem of Damanik and Lenz, we show that there is no subexponentially growing sequence for which the spectra of all discrete Schrödinger operators associated with subshifts whose complexity grows faster than the given sequence have only finitely many gaps.

1. Boshernitzan’s complexity conditions

For a symbolic dynamical system , many of the isomorphism invariants we have are statements about the growth rate of the word complexity function , which counts the number of distinct cylinder sets determined by words of length having nonempty intersection with . For example, the exponential growth of is the topological entropy of , while the linear growth rate of gives an invariant to begin distinguishing between zero entropy systems. Of course there are different senses in which the growth of could be said to be linear and different invariants arise from them. For example, one can consider systems with linear limit inferior growth, meaning , or the stronger condition of linear limit superior growth, meaning . (There exist systems satisfying the first condition but not satisfying the second.)

Under the assumption of linear limit inferior growth, and with a further hypothesis that the system is minimal, Boshernitzan Reference 2 showed that the system only supports finitely many -invariant ergodic probability measures. Boshernitzan also considered another version of linear complexity on a minimal shift, studying linear measure growth, also referred to in the literature (see for example Reference 7) as condition (B): if is a -invariant Borel probability measure on , assume that there exists a sequence of integers such that

where denotes the measure of the cylinder set determined by the word and denotes the length of the word . Boshernitzan showed that linear measure growth implies that the minimal subshift is uniquely ergodic. Another consequence of linear measure growth, for word complexity, is that is finite.

Each of these three linear complexity assumptions, linear limit inferior growth, linear limit superior growth, and linear measure growth, immediately implies that the associated system has zero topological entropy. It is natural to ask which of these conditions imply any of the others. One of our main results is that while linear measure growth implies linear limit inferior growth, it does not imply linear limit superior growth. In fact, we show linear measure growth is flexible enough that examples satisfying it can be constructed with limit superior growth faster than any pre-assigned subexponential growth rate.

A second motivation for the construction we give comes from a question on the spectra of discrete Schrödinger operators that arise from a subshift. If is a shift, then each defines a discrete Schrödinger operator by

(and is called the potential function for this operator). Characterizing the spectra of discrete Schrödinger operators is an active field of study (e.g., Reference 1Reference 4Reference 7) and we refer the reader to Reference 5Reference 6 for excellent surveys on the theory of discrete Schrödinger operators associated with symbolic systems. For operators built in this way, the dynamical properties of can influence the spectral properties of for any . When is minimal, Damanik (personal communication) asked whether the condition that implies that the spectrum of can have only finitely many gaps. Our example shows that the assumption of positive entropy in this question cannot be relaxed to just ask that grow “nearly exponentially” infinitely often: for any subexponential rate our example, via a theorem of Damanik and Lenz Reference 7, gives a Schrödinger operator whose spectrum has infinitely many gaps and whose complexity is larger than infinitely often.

We turn to stating our main theorem. For a word in the language of a subshift , we denote the cylinder set starting at zero it determines by and we denote the words of length in the language of the subshift by (for further discussion of the definitions, see Section 2.1):

Theorem 1.1.

Let be a sequence of positive integers satisfying

There exists a minimal and uniquely ergodic subshift such that

and such that the unique invariant measure has the property that there is a sequence satisfying

The hypothesis in this theorem is a type of subexponential growth on the sequence and the constructed system is a zero entropy system satisfying the Boshernitzan condition while the factor complexity grows faster than the given sequence. To prove the theorem, it suffices to show that the system supports a measure satisfying the property Equation 2, as it then follows from Boshernitzan Reference 3, Theorem 1.2 that the system is uniquely ergodic.

An immediate corollary of Theorem 1.1, combined with a theorem of Damanik and Lenz Reference 7, Theorem 2, is the following:

Corollary 1.2.

Let be a sequence of positive integers which grows subexponentially in the sense of Equation 1. There exists a Cantor set , of Lebesgue measure zero, and a minimal subshift such that

and for every the discrete Schrödinger operator given by

has spectrum exactly .

2. Background

2.1. Symbolic systems

We work over the alphabet and consider . We denote as and we endow with the topology induced by the metric . The left shift is defined by for all . If is closed and -invariant, then is a subshift.

If , then the central cylinder set determined by is defined to be

If , then the one-sided cylinder set determined by is defined to be

If is a subshift and , the words of length are defined to be the collection of all such that , and the language of the subshift is the union of all the words:

If is a word, we say that is a subword of if for some (possibly empty) words .

For a subshift , the word complexity is defined to be the number of words of length in the language:

2.2. Well approximable irrationals

A key ingredient in our construction is Theorem 2.1 of V. Sos Reference 9 (formerly known as the Steinhaus Conjecture).

Theorem 2.1 (Three Gap Theorem).

Assume and , the partition of the unit circle determined by the points , with all points taken . Then the subintervals determined by this partition have at most three distinct lengths, and when there are three distinct lengths, the largest length is the sum of the other two.

Given an integer and irrational , we refer to the partition determined by the points of the unit circle as the -step partition, and make use of it for well chosen . An irrational real number is well approximable if there exists a sequence of integers such that for each , the associated -step partition in the Three Gap Theorem has three distinct lengths and the ratio of the smallest to the largest length in such a partition tends to zero as , and without loss of generality we can assume that shortest length in the -step partition is not present in the -step partition. (This sequence is obtained as the denominators in the regular continued fraction expansion of , and this can be rephrased as unbounded partial quotients.) Furthermore, we can choose the sequence such that the smallest length present in the -step partition is not present for in the -step partition.

An irrational that is not well approximable is said to be badly approximable, and the set of badly approximable reals has Lebesgue measure zero. Notice that if is well approximable and is the associated sequence, then the -step partition in the Three Gap Theorem has only two distinct lengths and the ratio of their lengths tends to as .

2.3. Sturmian systems

To make use of the approximations determined by the Three Gap Theorem, we use Sturmian sequences. To define this notion, let be an irrational real number and consider the partition of and let denote the rotation . For any and each , define

Let be closure of the set of all sequences of the form

Then is called the Sturmian shift with rotation angle . A classical fact is that the system is minimal, uniquely ergodic, and for all .

Moreover, words correspond to the cells of , and with respect to the unique invariant measure , the measure of the cylinder set is the Lebesgue measure of the cell of corresponding to . In other words, there is a bijection

such that for any word , we have

In view of the discussion in Section 2.2, if is well approximable, there exists a sequence such that

Recall that is uniquely ergodic if there exists a unique Borel probability -invariant measure on . Recasting this definition in terms of the language, the subshift is uniquely ergodic if and only if for any , there exists such that for any there is an integer with the property that for all with , we have

In this case, is the measure of the cylinder set with respect to the unique invariant measure on .

3. The construction

We construct a minimal subshift such that

and for which there exists an invariant measure supported on and a sequence satisfying

3.1. Setup

We fix (any value suffices) and choose a well approximable real number satisfying

Let denote the Sturmian shift with rotation angle and let denote the (unique) invariant measure supported on (see Section 2.3). For each , let denote the partition of into subintervals whose endpoints are given by the set

where, as usual, all points are taken in , meaning modulo .

Using Equation 4 derived from the well approximability of , there exists satisfying

(in fact there exist infinitely many such ). The partition is obtained from the partition by subdividing one of the subintervals in into two pieces. Thus the length of the longest subinterval in is at most twice the length of the longest subinterval in . Similarly the length of the shortest subinterval in is at least as long as the length of the shortest subinterval in . Therefore we also have

We are now ready to begin our construction.

and let and denote the bijections defined in Equation 3. Then

and

Since is uniquely ergodic, we can choose such that for any and any word and any word , we have

(here we use the fact that is minimal and so ). Since is Sturmian, we have for all . Equivalently, this means that for all . In particular, for all there is a unique word for which both and are elements of . Let be the unique word of length with this property and let be the unique word of length with this property. Note that for any , the unique word with this property has as its rightmost subword of length and as its rightmost subword of length . (Note that we refer to subwords as being right or left in another word, where this is meant in the natural sense of ordering: a subword is the rightmost subword of the word if for some word , with the analogous interpretation for other directional indicators.)

Since is minimal and not periodic, all sufficiently long words in contain every word of length (which is ) as a subword and there is a uniform gap (which depends only on ) between consecutive occurrences of any word in , and by assuming that we consider sufficiently long words, we can also assume that none of the words has period at most . Let be sufficiently large such that this holds (thus we have a uniform gap between occurrences of any word in and no words of length have period at most ) and such that the unique word for which both and are in has this property. Then the rightmost subword of of length is and there is an occurrence of within distance of the left edge of . Define to be the subword of that begins with the leftmost occurrence of and ends just before the rightmost occurrence of (meaning we remove the rightmost letters of to obtain the end of the word ). Note that and so Equation 12 holds for all words in and (because its length is at least ). Since , every word in occurs as a subword of . Moreover every subword of of length is an element of , since and the leftmost subword of length in is . Since is aperiodic, there exists an integer such that

and assume that is sufficiently large such that no word of length can be concatenated with itself times and still be in . Set

Let and let be the unique word for which both and are elements of . Since , every element of occurs as a subword of . Let be the subword of that begins at the leftmost occurrence of and ends just before the rightmost occurrence of . Then and so Equation 12 holds for the word when taking . Since , every word in occurs as a subword of , Moreover, every subword of length that occurs in , , and is in , since , the leftmost subword of is , and the leftmost subword of is . Finally we define two words:

By construction both of these words are periodic and we let denote the minimal period of and let denote the minimal period of . These words have the following properties:

(1)

;

(2)

does not occur as a subword of . Namely since , , and so an occurrence of would occur as a subword of . Since the word repeated times and so an occurrence of in would force the word (repeated times) to occur as a subword of , contradicting the fact that (repeated times) is not in ;

(3)

does not occur as a subword of (because and if occurred in it would force an occurrence of in );

(4)

does not occur as a subword of (again because such an occurrence would force an occurrence of in );

(5)

occurs exactly once as a subword of . To check this, note that cannot overlap by , or more, symbols without forcing an occurrence of in . So an occurrence of (the word) must overlap (the leftmost subword of ) on more than symbols. This means that the occurrence of (the word) has to be offset from the beginning of by a multiple of . If this occurrence of overlaps by at least many symbols, then since is a multiple of (where is the length of the leftmost in ), and (now the hypothetical nontrivial occurrence of in ) is offset from the beginning of by a multiple of , this implies that the leftmost letters of must agree with the leftmost letters of . But begins with the word and begins with the word , a contradiction. This means that (the subword) overlaps on at most symbols. and so this occurrence of is offset from the beginning of by at most symbols, which implies that . But is the concatenation of copies of and the length , and so has period bounded by the length . Since , we have a contradiction since no word this long in this language (by definition of ) has period less than or equal to the length .

Thus we cannot have both and occurring as subwords of , contradicting the fact that is a subword of , is a subword of , and every word in occurs as a subword of .

Next we define two more words, and , as follows:

Note that all of the words , , , and contain at least once as a subword. Consider where such a subword could occur:

We analyze where it can occur in , and the analysis for the other three cases is similar. Since does not occur as a subword of , the prefix (in ) cannot completely overlap the leftmost in . This means that the farthest to the left that this prefix can occur is if it begins one letter after the beginning of the leftmost in . But since the word in cannot be completely contained in the central of , the farthest to the left can occur in is to have the at least partially overlap the rightmost in . Also, since the only place in that can occur is at the leftmost edge, the in cannot occur anywhere farther to the right in than the rightmost (otherwise it would force an occurrence of in which would guarantee that one of the subwords in exactly overlaps the rightmost in , which is impossible since ). Therefore any occurrence of in must have the in partially overlap the rightmost in , but not extend any farther to the right than this occurrence of . If it did not exactly overlap the rightmost in , then the immediately preceding the rightmost (in ) occurs in a nontrivial place within the rightmost in , a contradiction.

Lemma 3.1.

Any element that can be written as a bi-infinite concatenation of the words and can be written in a unique way as such a concatenation.

A shift with this property is sometimes known as a uniquely decipherable coded shift.

Proof.

Note that and have the same length and are not the same word. We have already noted that the word occurs in each of , , , and and, moreover, it occurs exactly once in each such word. If can be written as a bi-infinite concatenation of the words and , then there must be an occurrence of within distance of the origin. Choose a way to write as a concatenation of and and mark the locations in where this choice places the beginnings of these words. Find an occurrence of within distance of the origin. Since , this occurrence must be contained in one of the words , , , or that begins from our marked set of integers. But occurs exactly once in any such word and its location is always exactly symbols from where the word started. This allows us to determine where the marked integers in this occurrence of , , , or are located. In turn, this means that we can read off the sequence of words and that were concatenated to produce by starting from one of the marked integers and looking at blocks of size moving to the right and left.

Thus, once we find an occurrence of within distance of the origin in , the locations (in ) where the words and begin are determined, and once these locations are determined, the bi-infinite sequence of and is also determined. In other words, there is a unique way to write as such a bi-infinite concatenation.

Lemma 3.2.

Let be the subshift consisting of all elements of that can be written as bi-infinite concatenations of the words and . Let be any subshift of and let be any -invariant probability measure on . Recall that is defined in Equation 9 and is fixed to guarantee that Equation 12 holds. Then for any we have and we have

and

Proof.

By construction of the words and (from which and are built), the claim about equality of the languages holds because and every subword of length that occurs in any of , , , or is in the language . To see the claim about measure, by the ergodic decomposition theorem it suffices to show the analogous claim holds when is replaced by any ergodic measure supported on . Let be fixed. Fix an ergodic measure and fix a generic element . Let denote the indicator function of the cylinder set determined by . Then for all sufficiently large , we have

We know that can be parsed into an infinite concatenation of the words and . Fixing , there exists a subword of whose length is at least that is a concatenation of the words and . Let be the indices where this subword starts and ends. Then

The second and third of these averages tend to zero as , and so for sufficiently large we can assume they are both at most . The first average can be rewritten as:

Since , , and since any subword of length that occurs in any of , , , and is in the language , then for each fixed we have

by Equation 12. Therefore, recalling that is sufficiently large, we have that

Combining this with Equation 13, it follows that

By Equation 10 and Equation 11 we know that

and

Thus we have

since . Similarly,

Lemma 3.3.

The shift defined in Lemma 3.2 satisfies

Proof.

The number of distinct words whose length is any particular multiple of is at least and at most : as a word of this length must contain an occurrence of , this identifies how this word is parsed as a subword in the concatenation of the words and (other than perhaps the leftmost and rightmost words in the concatenation).

We fix a subexponentially growing sequence . It follows from Lemma 3.3 that

for all but finitely many . Thus,

we can choose some such that , and then fix two (distinct) words , of equal length, that each contain every element of as a subword and which can be written as a concatenation of the words and . Finally define

Arguing as with words that can be written as bi-infinite concatenations of and , observe that any element of that can be written as a bi-infinite concatenation of and can be written in a unique way as such a concatenation. Let be the subshift consisting of all elements of that can be written as a bi-infinite concatenation of the word and . Then , meaning Lemma 3.2 applies to any -invariant probability measure on . Furthermore, by Equation 14, we have that

3.2. Inflated subshifts

The subshift constructed in the last paragraph of Section 3.1 and satisfying Equation 18 consists of all bi-infinite concatenations of the words and . In this way we can think of has being a “full shift” on the “symbols” and . Since and are actually words, rather than symbols, we prove two lemmas that show nevertheless does have some properties that a full shift would have, such as a unique representation of every element of as a concatenation of and . In what follows, we abstract the construction so that it can then be applied in an inductive construction. Note that if the words and in Lemma 3.4 are taken to be the words and from Section 3.1, then and would be exactly the words and defined in Equation 16 and Equation 17.

Lemma 3.4.

Assume that are two distinct words of equal length. Let

Then for any that can be written as a bi-infinite concatenation of the words and , there is a unique way to write it as such a concatenation. Furthermore, for such an and for , whenever occurs as a subword of , there exists a sequence …, , , , , , … such that is (a finite shift of) the sequence

Proof.

Let be such that it can be written as a bi-infinite concatenation of the words and ; choose some way to do this. Since and have equal length, there exists such that, when is written as a bi-infinite concatenation of and , all of the concatenated words begin at indices of that are congruent to modulo . Now fix and suppose occurs as a subword of , beginning at index .

If , then we have already found the decomposition appearing in Equation 19. We aim to show it is not possible that , which implies the uniqueness of the decomposition of into a bi-infinite concatenation of and and also provides the decomposition appearing in Equation 19. Note that our chosen way to write as a bi-infinite concatenation of and implies that there exist such that arises as a subword of in a location that is neither the leftmost nor the rightmost.

Let be the periodic word

Then is a period of . We claim it is the minimal period of . To see this, note that the minimal period of must divide . If the minimal period of is , then the leftmost subword of length occurring in is equal to the rightmost subword of and the period of implies that , a contradiction. If the minimal period of is , then it is immediate that and again we have a contradiction. Therefore the minimal period of is at most . The minimal period of is also a period of the word , but since is also a period of , the Fine-Wilf Theorem Reference 8 implies that the minimal period of divides . Therefore and we have a contradiction. Thus, the minimal period of is .

Now, occurs as a subword of in a location that is neither the leftmost nor the rightmost. This occurrence of overlaps at least one of and (in their obvious positions within ) on at least letters. We analyze the case when overlaps on at least of its letters (the other case is similar). Note that and are both comprised of seven blocks of length and both start with and end with . In between these common prefixes and suffixes, has the block followed by and has the block . In both of these words, we refer to the word and , respectively, as their “block of ’s.” Note that the word contains the word ( self-concatenated five times) beginning to the right of the block of ’s in . The block of ’s in does not occur as a subword of : namely, since we have shown that the minimal period of is , and since the minimal period of is which is a proper divisor of , the word cannot occur as a subword of . In fact, in the case that , the rightmost in does not occur in and so the location where occurs in is far enough to the left that the block of ’s in extends by at most to the right of the block of ’s in . This means the suffix in completely overlaps (meaning it contains the occurrence of) the suffix in . Since the minimal period of is , this guarantees that the word appears in a multiple of letters from the left. If it appears exactly letters from the left, then the leftmost in the block of ’s of overlaps the word in , implying that and we have a contradiction. Otherwise the leftmost in the block of ’s of appears as a subword of in , again implying that and a contradiction. Therefore cannot have occurred in in any location other than the leftmost or the rightmost.

Lemma 3.5 shows that if we pick a subshift which has desirable ergodic properties, and “inflate” it by making the replacements , and taking the orbit closure of all elements of obtained in this way, then the “inflated” shift also has desirable ergodic properties.

Lemma 3.5.

Let be a subshift. Assume there exist an integer and a positive constant such that for any ergodic measure supported on , we have

Assume that are two distinct words of equal length and set

Let be the subshift consisting of all elements of that can be written as a bi-infinite concatenation

where . If is any ergodic measure supported on , then

Proof.

Let be any fixed element and let . Then

Let be an ergodic measure supported on and let be a generic point for the measure . Choose such that

and

Since is ergodic,

and

We analyze occurrences of in , noting that the same analysis applies for occurrences of . Any occurrence of in occurs in a concatenation of the words and , some number of times. Since and , whenever occurs it must completely contain at least one of the words , , , or as a subword.

Any occurrence of in occurs in a concatenation of words which are all and . Since , at least two of these concatenated words occur as a subword of and so the location of in the concatenation the words and is uniquely determined by Lemma 3.4. Thus each occurrence of occurs either as an exact concatenation of many of the words and , or as a subword of a concatenation of exactly of the words and (which is neither the leftmost nor rightmost subword of length ) and all but (perhaps) the first and last of the words and can be determined from the word . In the first case, when is itself an exact concatenation of of the words and , we adopt the convention of viewing it as a subword of the concatenation of many of and by appending the extra or onto its right when it occurs. Therefore there are at most four ways to concatenate the words and such that occurs as a subword, corresponding to the ambiguity of the edge (first and last) concatenated words and that there are at most two choices for each of these edge words, or the ambiguity of the rightmost word when is an exact concatenation of of the words and . This means that the asymptotic frequency with which occurs as a subword of is times the sum of the measures of the cylinder sets of length (in the shift ) that determine occurrences of in . But by Equation 21, the frequency with which this cylinder set of length occurs (in ) is at most . Therefore

Similarly we have

where this time there is at least one word which is a concatenation of exactly many of the words and that imply an occurrence of . Therefore, we have

3.3. Induction

Let be a subexponentially growing sequence of positive integers, meaning

We inductively construct a sequence of shifts , , , …and ultimately define our subshift from Theorem 1.1. Let be the smallest integer that satisfies Equation 8 and let, by taking in Equation 9, be the subshift constructed at the end of Section 3.1 (where it was called ). Let be the parameter arising in Section 3.1 and let and be the words defined in Equations Equation 16 and Equation 17 constructed from the sequence and in Equation 15 and let be the parameter in the sentence following Equation 15. Then we have .

Now suppose we have constructed a nested sequence of subshifts

a sequence of positive integers , and a sequence of words , , , , …, , where , are two distinct words of equal length (and this common length is at least ). We suppose that for each , is the subshift obtained by taking all possible bi-infinite concatenations of the words and . Suppose further that for any and any we have and

for any ergodic measure supported on . Take to be an integer satisfying Equation 8 and let when is chosen to be in Equation 9. We produce the subshift , constructed in Section 3.1 and the words and with this choice of . Next, we apply Lemma 3.5 with , , , , and to produce a new subshift . Note that since consisted of all possible concatenations of and , it follows that consists of all possible concatenations of the “inflated” versions of the words and , where and . We define and to be the words and after this replacement is applied. Note that every element of can be written as a bi-infinite concatenation of and , and so .

Note that since , by Equation Equation 18 and taking , we have guaranteed that for all . Finally, by Lemmas 3.2 and 3.5, we have

for all .

Finally, define

Then since the subshifts are nested, is nonempty. Since each pattern occurs syndetically, the system is minimal. Finally, by construction, we obtain a subshift satisfying for all , in particular

and such that

for all . This concludes the proof of Theorem 1.1.

Acknowledgments

We thank David Damanik for bringing this question to our attention and for helpful remarks during the preparation of this paper, and we thank the referee for careful reading that significantly improved our paper. The first author thanks Northwestern for its hospitality while this work was completed.

Mathematical Fragments

Theorem 1.1.

Let be a sequence of positive integers satisfying

There exists a minimal and uniquely ergodic subshift such that

and such that the unique invariant measure has the property that there is a sequence satisfying

Theorem 2.1 (Three Gap Theorem).

Assume and , the partition of the unit circle determined by the points , with all points taken . Then the subintervals determined by this partition have at most three distinct lengths, and when there are three distinct lengths, the largest length is the sum of the other two.

Equation (3)
Equation (4)
Equation (8)
Equation (9)
Equation (10)
Equation (11)
Equation (12)
Lemma 3.2.

Let be the subshift consisting of all elements of that can be written as bi-infinite concatenations of the words and . Let be any subshift of and let be any -invariant probability measure on . Recall that is defined in Equation 9 and is fixed to guarantee that Equation 12 holds. Then for any we have and we have

and

Equation (13)
Lemma 3.3.

The shift defined in Lemma 3.2 satisfies

Equation (14)
Equation (15)
Equations (16), (17)
Equation (18)
Lemma 3.4.

Assume that are two distinct words of equal length. Let

Then for any that can be written as a bi-infinite concatenation of the words and , there is a unique way to write it as such a concatenation. Furthermore, for such an and for , whenever occurs as a subword of , there exists a sequence …, , , , , , … such that is (a finite shift of) the sequence

Lemma 3.5.

Let be a subshift. Assume there exist an integer and a positive constant such that for any ergodic measure supported on , we have

Assume that are two distinct words of equal length and set

Let be the subshift consisting of all elements of that can be written as a bi-infinite concatenation

where . If is any ergodic measure supported on , then

Equation (21)

References

Reference [1]
Artur Avila and Raphaël Krikorian, Reducibility or nonuniform hyperbolicity for quasiperiodic Schrödinger cocycles, Ann. of Math. (2) 164 (2006), no. 3, 911–940, DOI 10.4007/annals.2006.164.911. MR2259248,
Show rawAMSref \bib{Avila-Krikorian}{article}{ author={Avila, Artur}, author={Krikorian, Rapha\"{e}l}, title={Reducibility or nonuniform hyperbolicity for quasiperiodic Schr\"{o}dinger cocycles}, journal={Ann. of Math. (2)}, volume={164}, date={2006}, number={3}, pages={911--940}, issn={0003-486X}, review={\MR {2259248}}, doi={10.4007/annals.2006.164.911}, }
Reference [2]
Michael Boshernitzan, A unique ergodicity of minimal symbolic flows with linear block growth, J. Analyse Math. 44 (1984/85), 77–96, DOI 10.1007/BF02790191. MR801288,
Show rawAMSref \bib{boshernitzan}{article}{ author={Boshernitzan, Michael}, title={A unique ergodicity of minimal symbolic flows with linear block growth}, journal={J. Analyse Math.}, volume={44}, date={1984/85}, pages={77--96}, issn={0021-7670}, review={\MR {801288}}, doi={10.1007/BF02790191}, }
Reference [3]
Michael D. Boshernitzan, A condition for unique ergodicity of minimal symbolic flows, Ergodic Theory Dynam. Systems 12 (1992), no. 3, 425–428, DOI 10.1017/S0143385700006866. MR1182655,
Show rawAMSref \bib{boshernitzan2}{article}{ author={Boshernitzan, Michael D.}, title={A condition for unique ergodicity of minimal symbolic flows}, journal={Ergodic Theory Dynam. Systems}, volume={12}, date={1992}, number={3}, pages={425--428}, issn={0143-3857}, review={\MR {1182655}}, doi={10.1017/S0143385700006866}, }
Reference [4]
J. Bourgain and S. Jitomirskaya, Absolutely continuous spectrum for 1D quasiperiodic operators, Invent. Math. 148 (2002), no. 3, 453–463, DOI 10.1007/s002220100196. MR1908056,
Show rawAMSref \bib{Bourgain-Jitomirskaya}{article}{ author={Bourgain, J.}, author={Jitomirskaya, S.}, title={Absolutely continuous spectrum for 1D quasiperiodic operators}, journal={Invent. Math.}, volume={148}, date={2002}, number={3}, pages={453--463}, issn={0020-9910}, review={\MR {1908056}}, doi={10.1007/s002220100196}, }
Reference [5]
David Damanik, Strictly ergodic subshifts and associated operators, Spectral theory and mathematical physics: a Festschrift in honor of Barry Simon’s 60th birthday, Proc. Sympos. Pure Math., vol. 76, Amer. Math. Soc., Providence, RI, 2007, pp. 505–538, DOI 10.1090/pspum/076.2/2307746. MR2307746,
Show rawAMSref \bib{Damanik2}{article}{ author={Damanik, David}, title={Strictly ergodic subshifts and associated operators}, conference={ title={Spectral theory and mathematical physics: a Festschrift in honor of Barry Simon's 60th birthday}, }, book={ series={Proc. Sympos. Pure Math.}, volume={76}, publisher={Amer. Math. Soc., Providence, RI}, }, date={2007}, pages={505--538}, review={\MR {2307746}}, doi={10.1090/pspum/076.2/2307746}, }
Reference [6]
David Damanik, Schrödinger operators with dynamically defined potentials, Ergodic Theory Dynam. Systems 37 (2017), no. 6, 1681–1764, DOI 10.1017/etds.2015.120. MR3681983,
Show rawAMSref \bib{Damanik}{article}{ author={Damanik, David}, title={Schr\"{o}dinger operators with dynamically defined potentials}, journal={Ergodic Theory Dynam. Systems}, volume={37}, date={2017}, number={6}, pages={1681--1764}, issn={0143-3857}, review={\MR {3681983}}, doi={10.1017/etds.2015.120}, }
Reference [7]
David Damanik and Daniel Lenz, A condition of Boshernitzan and uniform convergence in the multiplicative ergodic theorem, Duke Math. J. 133 (2006), no. 1, 95–123, DOI 10.1215/S0012-7094-06-13314-8. MR2219271,
Show rawAMSref \bib{DL}{article}{ author={Damanik, David}, author={Lenz, Daniel}, title={A condition of Boshernitzan and uniform convergence in the multiplicative ergodic theorem}, journal={Duke Math. J.}, volume={133}, date={2006}, number={1}, pages={95--123}, issn={0012-7094}, review={\MR {2219271}}, doi={10.1215/S0012-7094-06-13314-8}, }
Reference [8]
N. J. Fine and H. S. Wilf, Uniqueness theorems for periodic functions, Proc. Amer. Math. Soc. 16 (1965), 109–114, DOI 10.2307/2034009. MR174934,
Show rawAMSref \bib{FW}{article}{ author={Fine, N. J.}, author={Wilf, H. S.}, title={Uniqueness theorems for periodic functions}, journal={Proc. Amer. Math. Soc.}, volume={16}, date={1965}, pages={109--114}, issn={0002-9939}, review={\MR {174934}}, doi={10.2307/2034009}, }
Reference [9]
V. Sos, On the distribution of the sequence , Ann. Univ. Sci. Budapest, Eötvös Sect. Math. 1 (1958), 127–134.

Article Information

MSC 2020
Primary: 37B10 (Symbolic dynamics)
Secondary: 37B40 (Topological entropy), 35J10 (Schrödinger operator, Schrödinger equation), 37A35 (Entropy and other invariants, isomorphism, classification in ergodic theory)
Keywords
  • Subshift
  • complexity
  • Schrödinger operator
  • uniquely ergodic
Author Information
Van Cyr
Department of Mathematics, Bucknell University, 380 Olin Science Building, Lewisburg, Pennsylvania, 17837
van.cyr@bucknell.edu
MathSciNet
Bryna Kra
Department of Mathematics, Northwestern University, 2033 Sheridan Road, Evanston, IL, 60208
kra@math.northwestern.edu
ORCID
MathSciNet
Additional Notes

The second author was partially supported by NSF grant DMS-1800544.

Communicated by
Katrin Gelfert
Journal Information
Proceedings of the American Mathematical Society, Series B, Volume 9, Issue 11, ISSN 2330-1511, published by the American Mathematical Society, Providence, Rhode Island.
Publication History
This article was received on , revised on , , and published on .
Copyright Information
Copyright 2022 by the authors under Creative Commons Attribution-NonCommercial 3.0 License (CC BY NC 3.0)
Article References
  • Permalink
  • Permalink (PDF)
  • DOI 10.1090/bproc/90
  • MathSciNet Review: 4467310
  • Show rawAMSref \bib{4467310}{article}{ author={Cyr, Van}, author={Kra, Bryna}, title={Boshernitzan's condition, factor complexity, and an application}, journal={Proc. Amer. Math. Soc. Ser. B}, volume={9}, number={11}, date={2022}, pages={95-110}, issn={2330-1511}, review={4467310}, doi={10.1090/bproc/90}, }

Settings

Change font size
Resize article panel
Enable equation enrichment

Note. To explore an equation, focus it (e.g., by clicking on it) and use the arrow keys to navigate its structure. Screenreader users should be advised that enabling speech synthesis will lead to duplicate aural rendering.

For more information please visit the AMS MathViewer documentation.